Statement and Argument Questions and Answers
question divider line

Home > Verbal Tests > Statement and Argument > Statement and Argument Questions
question divider line

Each question given below consists of a statement, followed by two arguments numbered I and II. You have to decide which of the arguments is a 'strong' argument and which is a 'weak' argument.


Give answer:


* (A) If only argument I is strong

* (B) If only argument II is strong

* (C) If either I or II is strong

* (D) If neither I nor II is strong and

* (E) If both I and II are strong.

Statement: Should all the unauthorized structures in the city be demolished?

 

Arguments:

 

1. No. Where will the people residing in such houses live?

2. Yes. This will give a clear message to general public and they will refrain from constructing unauthorized buildings.


A. Only argument I is strong
B. Only argument II is strong
C. Either I or II is strong
D. Neither I nor II is strong
E. Both I and II are strong

Answer & Explanation:

Answer: Option B

Explanation: The demolition of unauthorized buildings would teach a lesson to the unscrupulous builders and also serve as a warning for the citizens not to indulge in such activities in the future. This is essential, as unauthorized constructions impose undue burden on the city's infrastructure. So, only argument II holds strong.



question divider line

Statement: Should there be a maximum limit for the number of ministers in the Central Government?

 

Arguments:

 

1. No. The political party in power should have the freedom to decide the number of ministers to be appointed.

2. Yes. The number of ministers should be restricted to a certain percentage of the total number of seats in the parliament to avoid unnecessary expenditure.


A. Only argument I is strong
B. Only argument II is strong
C. Either I or II is strong
D. Neither I nor II is strong
E. Both I and II are strong

Answer & Explanation:

Answer: Option B

Explanation: Clearly, there should be some norms regarding the number of ministers in the Government, as more number of ministers would unnecessarily add to the Government expenditure. So, argument II holds strong; Also, giving liberty to the party in power could promote extension of unreasonable favour to some people at the cost of government funds. So, argument I does not hold.



question divider line
question divider line

Statement: Should foreign films be banned in England?

 

Arguments:

 

1. Yes. They depict an alien culture which adversely affects our values.

2. No. Foreign films are of a high artistic standard.


A. Only argument I is strong
B. Only argument II is strong
C. Either I or II is strong
D. Neither I nor II is strong
E. Both I and II are strong

Answer & Explanation:

Answer: Option D

Explanation: Clearly, foreign films depict the alien culture but this only helps in learning more. So, argument I does not hold. Also, the reason stated in argument II is not strong enough in contradicting the ban. So, it also does not hold.



question divider line

Statement: Is buying things on instalments profitable to the customer?

 

Arguments:

 

1. Yes. He has to pay less.

2. No, paying instalments upsets the family budget.


A. Only argument I is strong
B. Only argument II is strong
C. Either I or II is strong
D. Neither I nor II is strong
E. Both I and II are strong

Answer & Explanation:

Answer: Option D

Explanation: In buying things on instalments, a customer has to pay more as the interest is also included. So, argument I does not hold. Moreover, one who buys an item on instalments maintains his future budget accordingly as he is well acquainted with when and how much he has to pay, beforehand. So, argument II is also not valid.



question divider line

Statement: Should adult education programme be given priority over compulsory education programme?

 

Arguments:

 

1. No. It will also help in success of compulsory education programme.

2. Yes. It will help to eliminate the adult illiteracy.


A. Only argument I is strong
B. Only argument II is strong
C. Either I or II is strong
D. Neither I nor II is strong
E. Both I and II are strong

Answer & Explanation:

Answer: Option B

Explanation: Clearly, argument I gives a reason in support of the statement and so it does not hold strong against it. The adult education programme needs to be given priority because it shall eliminate adult illiteracy and thus help in further spread of education. So, only argument II is strong enough.



question divider line

Page 1   | Page 2   | Page 3   | Page 4   | Page 5   | Page 6   | Page 7   | Page 8   | Page 9  

question divider line

More educational and fun tests below.




Questions or comments? Please discuss below.




Copyright © The Online Test Centre - Since 2005 | Terms and Conditions & Privacy Policy | Blog


The largest provider of tests on the web. We have thousands of questions covering

 

numerical and verbal aptitude tests, diagrammatic and logic tests, reasoning tests,

 

puzzles, general knowledge test questions and GMAT test advice and questions.

 

No login required. All tests are free. All the tests you need when you need them.